What is the correct angle of displacement for a clock arm?

In summary: It doesn’t, true, but the preceding three questions all use Cartesian...And they were all about something that would have a coordinate system, unlike a clock.And they were all about something that would have a coordinate system, unlike a clock.I am sorry but I am not clear with it. Can you elaborate it?I am sorry but I am not clear with it. Can you elaborate it?Sure. The previous three questions in the thread were about something that would have a coordinate system, such as an x-y plot. In those cases, it makes sense to use Cartesian coordinates for the calculation. But in the case of a clock, since it already has its own coordinate system (12, 3, 6,
  • #1
rudransh verma
Gold Member
1,067
95
Homework Statement
The minute hand of a wall clock measures 12cm from its tip to axis. The magnitude and angle of the displacement vector of tip are to be calculated for three intervals. What are the A) magnitude and B) angle from quarter after the hour to half past, C) for next half hour and D) for the hour after that?
Relevant Equations
Delta r= r2-r1
I calculated the angle as 225 whereas answer says something else.
 

Attachments

  • image.jpg
    image.jpg
    76.5 KB · Views: 109
  • image.jpg
    image.jpg
    50.9 KB · Views: 103
Physics news on Phys.org
  • #2
Please show your work, and please re-post your image rightside-up and scanned at higher quality so we can read it. Thank you.
 
  • Like
Likes Vanadium 50
  • #3
berkeman said:
Please show your work, and please re-post your image rightside-up and scanned at higher quality so we can read it. Thank you.
Edited
 
  • #4
rudransh verma said:
I calculated the angle as 225 whereas answer says something else.
Assuming:
- the +x-axis runs from the dial’s centre in the direction of 3 o’clock (for a 12 hour clock);
- angles are measured anticlockwise relative to the +x axis;
then I agree: the angle of displacement of the tip from quarter past the hour to half past the hour is 225º.

What is the ‘official’ answer?

Also, note that your homework statement in Post #1, has 4 parts (labelled A to D). But the image of the question from the textbook shows 6 parts (labelled a to f).
 
  • #5
Steve4Physics said:
What is the ‘official’ answer?
You can follow in image numbering.
 

Attachments

  • BFDFDD33-B230-45AD-A341-B9371AE5B2B4.jpeg
    BFDFDD33-B230-45AD-A341-B9371AE5B2B4.jpeg
    27 KB · Views: 88
  • #6
Here are some comments on the ‘official’ answers in the Post #5 attachment. Just my opinions though.

The units for the magnitudes of displacements should be cm. The units are missing (a very basic mistake!).

a) ##12\sqrt 2## should be ##12\sqrt 2## cm.

b) 135º only makes sense if the angle is measured anticlockwise from the 12 o'clock direction.

c) ##24## should be ##24## cm.

d) If the angles are measured as for part b) (above), then the answer should be 0º, not 180º.

e) Answer ‘e’ is mislabelled ‘f’ but is correct.

f) Answer ‘f’ is mislabelled ‘g’. Since the displacement has zero magnitude we should really say ‘θ is undefined’, not θ = 0.

What text-book/source are these questions/answers from?
 
  • Like
Likes hmmm27
  • #7
Steve4Physics said:
135º only makes sense if the angle is measured anticlockwise from the 12 o'clock direction.
This is the fig. The angle should be -135.
Also it is coming -45. Is it right? Because we take - angle when measuring clockwise.

Also this 180 degree doesn’t make sense. Because we measure angles from +x axis
 

Attachments

  • image.jpg
    image.jpg
    33.2 KB · Views: 80
  • #8
Steve4Physics said:
What text-book/source are these questions/answers from?
Principles of physics Resnik. 2D motion
 
  • #9
@berkeman can you clear my doubt in post 7
 
  • #10
Steve4Physics said:
135º only makes sense if the angle is measured anticlockwise from the 12 o'clock direction.
Or if measured clockwise from 3 o'clock.
Steve4Physics said:
d) If the angles are measured as for part b) (above), then the answer should be 0º, not 180º.
I make it 90° if measured anticlockwise from 3 o'clock. The given answer seems to be the rotational angle undergone by the hand, not the orientation of the displacement vector.
 
  • #11
haruspex said:
Or if measured clockwise from 3 o'clock.
Every angle of the displacement will be measured from 3 or +x axis. @Steve4Physics is walking on different path.
haruspex said:
I make it 90° if measured anticlockwise from 3 o'clock.
For the first sweep the angle should be -135. For second and third it should be 90.
But the answer says different.
The answer given are measuring the sweep angle , the angle between the arms (well there are not two arms, you understand) but we have to find the angle of the displacement vector.
 

Attachments

  • 2C5CD4BB-2D74-47FD-8214-E36E50D6F2B0.jpeg
    2C5CD4BB-2D74-47FD-8214-E36E50D6F2B0.jpeg
    27 KB · Views: 68
  • #12
haruspex said:
The given answer seems to be the rotational angle undergone by the hand, not the orientation of the displacement vector.
Yeah!
What will be the displacement vector magnitude for the next half hour(6oclock to 12 o clock). I don’t understand the answer. There is no triangle formation here.
 
  • #13
rudransh verma said:
Yeah!
What will be the displacement vector magnitude for the next half hour(6oclock to 12 o clock). I don’t understand the answer. There is no triangle formation here.
There is a degenerate triangle, having sides 12, 12, 24. But that is not the reason they've written it that way. What seems to have been intended is ##|\vec r|=\sqrt{\vec r.\vec r}=\sqrt{24\hat y .24\hat y }=\sqrt{24^2}##.
 
  • #14
haruspex said:
There is a degenerate triangle, having sides 12, 12, 24.

what is a degenerate triangle?
haruspex said:
What seems to have been intended is |r→|=r→.r→=24y^.24y^=242.
I don't understand the math
 
  • #15
rudransh verma said:
what is a degenerate triangle?
One in which the long side equals the sum of the other two. It collapses to a straight line.
 
  • #16
haruspex said:
One in which the long side equals the sum of the other two. It collapses to a straight line.
Ok. what is it with the square root of dot product? What are you doing there?
 
  • #17
rudransh verma said:
Ok. what is it with the square root of dot product? What are you doing there?
That is how the modulus of a vector is defined: ##|\vec r|=\sqrt{\vec r.\vec r}##.
 
  • #18
haruspex said:
That is how the modulus of a vector is defined: ##|\vec r|=\sqrt{\vec r.\vec r}##.
Modulus is defined as r=under root(r1^2 +r2^2).
In which book it is defined like this?
 
  • #21
rudransh verma said:
Seems higher grade stuff.
The entire question seems to be part of an introduction to vectors, and this definition of the modulus is fairly basic in that context.
The Wikipedia discussion may seem higher grade because it makes the distinction between vector spaces in general (which need not have anything corresponding to a dot product) and inner product spaces.
 
  • #22
Is there a definitive answer on the angles ? I get 135, 0 and not-applicable, based on the assumption that a "displacement vector" angle is an absolute to the coordinate system, and "12" on a clock-face seems a good place to have a 0/360.
 
Last edited:
  • #23
hmmm27 said:
I get 135, 0 and not-applicable
Where are you measuring the angles from? Isn't anticlockwise from the 3 o'clock position standard?
 
  • Like
Likes rudransh verma
  • #24
haruspex said:
Where are you measuring the angles from? Isn't anticlockwise from the 3 o'clock position standard?
The question doesn't mention Cartesian xy coordinate system. 2d polar, clockwise from noon, seemed appropriate for a clock.

For that matter, given precedent of WWII air-battle movies, I'd prefer to say 7:30, 12:00 and n/a.

Wonder where the OP sits on the second and third parts of the question.
 
  • #25
hmmm27 said:
The question doesn't mention Cartesian xy coordinate system.
It doesn’t, true, but the preceding three questions all use Cartesian coordinates.
 
  • #26
hmmm27 said:
Is there a definitive answer on the angles ? I get 135, 0 and not-applicable, based on the assumption that a "displacement vector"
I got -135, 90, 90 from x axis
 
  • #27
As
rudransh verma said:
I got -135, 90, 90 from x axis
has been pointed out a couple of times, the angle for the final case is indeterminate since the vector has zero magnitude.
 

1. What is the displacement of a clock arm?

The displacement of a clock arm refers to the change in position of the arm from its original position. It is usually measured in degrees or radians.

2. What causes the displacement of a clock arm?

The displacement of a clock arm is caused by the movement of the clock's mechanism, which includes gears and springs, that rotate the arm. It can also be caused by external factors such as being bumped or adjusting the time manually.

3. How is the displacement of a clock arm measured?

The displacement of a clock arm is measured by using a protractor or other measuring tools to determine the angle of rotation from the original position. This can also be calculated using the clock's timekeeping mechanism and the number of rotations it makes in a specific time period.

4. Can the displacement of a clock arm be changed?

Yes, the displacement of a clock arm can be changed by adjusting the clock's timekeeping mechanism. This can be done manually by using the clock's hands or by adjusting the internal gears and springs. However, the displacement will always be determined by the clock's mechanism and cannot be changed permanently.

5. What are the practical applications of understanding displacement of a clock arm?

Understanding the displacement of a clock arm is important in accurately telling time and keeping track of time intervals. It is also used in the study of physics and mechanics, as well as in the design and maintenance of clocks and other timekeeping devices.

Similar threads

  • Introductory Physics Homework Help
2
Replies
54
Views
2K
  • Introductory Physics Homework Help
Replies
12
Views
901
  • Introductory Physics Homework Help
Replies
2
Views
637
  • Introductory Physics Homework Help
Replies
11
Views
797
Replies
28
Views
884
  • Introductory Physics Homework Help
Replies
6
Views
743
  • Introductory Physics Homework Help
Replies
2
Views
118
  • Introductory Physics Homework Help
Replies
3
Views
1K
  • Introductory Physics Homework Help
Replies
2
Views
1K
Back
Top